Wrong MBE 7.7

¡Supera tus tareas y exámenes ahora con Quizwiz!

Under the authority of a federal voting rights statute, some states drew congressional districts in a manner calculated to increase the likelihood that members of historically disadvantaged minority racial groups would be elected. The U.S. Supreme Court declared these districts to be unconstitutional, as improper racial gerrymanders. In response to this ruling, Congress passed a new statute that explicitly denies the Supreme Court appellate jurisdiction over all future cases challenging the constitutionality of action taken under the authority of the federal voting rights statute. Which of the following is the most persuasive argument for the constitutionality of the new statute restricting the Supreme Court's appellate jurisdiction? A) Article III of the Constitution explicitly states that the Supreme Court's appellate jurisdiction is subject to such exceptions and regulations as Congress shall make B) The constitutional principle of separation of powers authorizes Congress to pass statutes calculated to reduce the effects of Supreme Court decisions that interfere with the exercise of powers that have been delegated to the legislative branch C) The establishment and apportionment of congressional districts directly affect interstate commerce, and the Constitution authorizes Congress to use its plenary authority over such commerce for nay purpose it believes will promote the general welfare D) The Fifteenth Amendment authorizes Congress to enforce the amendment's voting rights provisions by appropriate legislation, and Congress could reasonably determine that this restriction the Supreme Court's appellate jurisdiction is an appropriate means to that end.

(A) is correct. Article III explicitly gives Congress the power to make exceptions to the Supreme Court's appellate jurisdiction. (B) is incorrect. The separation of powers doctrine is described as a series of checks and balances among the three branches of government - just the opposite of what this choice suggests. (C) is incorrect. Although Congress has broad powers under the Commerce Clause, they are not without limit. The Commerce Clause gives Congress power to adopt laws concerning channels of interstate commerce, such as roads and airways; instrumentalities of interstate commerce, such as trucks and trains; and economic or commercial activities - even ones that take place solely within one state - that in aggregate have a substantial effect on interstate commerce. Congressional voting districts do not fall within any of these categories. Moreover, the General Welfare Clause is part of Congress's spending power (it may spend for the general welfare) and is not part of Congress's commerce power. (D) is incorrect because it is not as strong an answer as (A). Article III explicitly allows Congress to make exceptions to the Supreme Court's appellate jurisdiction without qualification. The enabling clause of the Fifteenth Amendment allows Congress to adopt legislation protecting the right to vote from discrimination. A law taking away from the Supreme Court jurisdiction to hear cases under a voting rights statute would seem to be the opposite of what is allowed by the Fifteenth Amendment's enabling clause.

A brick mason was hired by a builder under a written one-year contract, at an annual salary of $45,000, with employment to begin on March 1. Because the builder was unable to secure enough building contracts to keep all its employees busy during the season beginning March 1, it notified the brick mason on February 15 that it could not afford to employ him as a mason. At the same time, however, the builder offered to employ the mason, for the same contract period, as a night guard at an annual salary of $25,000. The mason declined the offer and remained unemployed during the year. No employment for brick masons was available in the community during the year, but the mason could have obtained other employment as a day laborer that would have paid up to $25,000 a year. At the end of the year, in an action against the builder for breach of contract, how much, if anything, is the mason entitled to recover? A) 20,000 B) 45,000 C) Nothing, because the builder did not act in bad faith when it discharged the mason D) Nothing, because the mason did not mitigate his damages

(B) is correct. When an employer breaches an employment contract, the standard measure of the employee's damages is the full contract price. However, the nonbreaching party cannot recover damages that could have been avoided with reasonable effort. If the breaching employer can prove that a comparable job in the same locale was available, then contract damages against that breaching employer for lost wages will be reduced by the wages that the plaintiff would have received from that comparable job. Here, there was no comparable employment available in the community for the mason. The only jobs available, a night guard or a day laborer, were not the same type of work as a brick mason and were at a significantly lower salary. (A) is incorrect. The mason was not obligated to accept employment in a different field at a much lower salary to mitigate his damages. (C) is incorrect. The builder is liable for damages because it breached the employment contract with the mason. The builder's reasons for breaching, even if in good faith, do not excuse the breach. (D) is incorrect. No comparable employment was available in the community during the year and, as stated above, the mason was not obligated to accept employment in a different field at a much lower salary to mitigate his damages.

A homeowner from State A hired a contractor from State B to build a vacation home for her in State C. The parties signed the contract in State A. The contractor breached the contract, and the homeowner sued the contractor in a court of State A, seeking damages of $100,000. The contractor removed the case to the federal court for State A. The homeowner promptly moved to remand the case to state court, arguing that venue was improper. Which of the following facts is most relevant to the court's decision on the homeowner's motion? A) The contract was signed in State A B) The contractor resides in State B C) The homeowner commenced the action in State A court D) The vacation home was to be built in State C

(C) The commencement of the action in State A is most relevant. When a case is removed from state court to federal court, venue is set in the federal district court that embraces the state court in which the action was pending, making the federal district court of State A the only appropriate venue choice. Thus, (C) is the correct answer because the most (indeed, only) relevant fact is the fact that the action was commenced in a court of State A. Unlike cases commenced in federal court, in cases removed to federal court, the residence of the defendant contractor (answer choice (B)) and the place where a substantial part of events relating to the claim occurred (answer choices (A) and (D)) are irrelevant.

A man decided to give a cabin he owned to his daughter at his death. To accomplish this goal, he delivered to his attorney a deed that fully complied with the applicable statute of frauds and told his attorney to record the deed when he died unless he later gave the attorney instructions to the contrary. Three weeks after dropping off the deed, the man properly drafted and executed his own will, which left all of his real property to his son. One year later, the man died, and the attorney immediately recorded the deed. At the time of the man's death, the cabin was titled in his name and he owned no other real property. The daughter and the son now disagree as to who is entitled to ownership of the cabin. Other than the jurisdiction's statute of frauds and statute of wills, there are no applicable statutes. Who is entitled to ownership of the cabin? A) The daughter, because the attorney was, for gift-law purposes, a trustee for the daughter B) The daughter, because the deed fully complied with the statute of frauds C) The son, because the deed was not delivered to the daughter during the man's lifetime D) The son, because the proper execution of the will revoked the earlier gift to the daughter

(C) is correct. A deed is not effective to transfer an interest in realty unless it has been delivered. Delivery refers to the grantor's intent. There must be words or conduct showing the grantor intended that the deed have some present operative effect- that title pass immediately and irrevocably, even though the right of possession may be postponed. If the right of possession is to be postponed until the grantor's death, the deed may be held "testamentary" and therefore void (unless executed with testamentary formalities). However, most courts hold that if the grantor executes a deed and gives it to another with instructions to give it to the grantee upon the grantor's death, the grantor's intent was to presently convey a future interest to the grantee (either a remainder, with a life estate reserved in the grantor, or an executory interest), and so the gift is inter vivos, not testamentary. To make an effective delivery, the grantor must relinquish absolute and unconditional control. That was not the case here. The man gave the deed to the attorney to record when the man died "unless he later gave the attorney instructions to the contrary." This language shows that the man did not relinquish control. Thus, there was no delivery, and the property passed by the will to the son. (A) is incorrect. A trustee holds legal title to property and has an equitable duty to deal with it for the benefit of the beneficiary. Here, the attorney was given a deed to hold and record at the appropriate time. The attorney never acquired title to the property, and the deed was in the daughter's name. Also, the attorney's duties were owed to the man; he had to hold the title and record it at the man's death unless instructed otherwise. (B) is incorrect. The statute of frauds is only one component of a valid transfer by deed. To transfer title, a deed must not only be in writing, but also be delivered. There must be words or conduct showing the grantor intended that the deed have some present operative effect-that title pass immediately and irrevocably, even though the right of possession may be postponed. Here, the man gave the deed to the attorney to record at his death "unless he gave the attorney instructions to the contrary." This language shows that the man did not relinquish control and thus there was no delivery. Without delivery, the daughter acquired no interest in the property. (D) is incorrect. The execution of a will has no effect on the testator's property because the will could be changed or revoked at any time until the testator dies. The will does not become effective until the testator dies. Thus, the execution of the man's will did not revoke the gift of the property to the daughter. Furthermore even an operative will cannot revoke a validly executed and delivered deed. A will has no effect on it because the testator no longer owns the property. Had the man delivered the deed without conditions, the property would belong to the daughter and would not pass by will to the son.

A pedestrian was injured when hit by a chair that was thrown from an upper-story hotel window. The pedestrian sued the occupants of all the rooms from which the chair might have been thrown. At trial, the pedestrian has been unable to offer any evidence as to the exact room from which the chair was thrown. The defendants have filed a motion for a directed verdict. Should the court grant the motion? A) No, because it is unreasonable to expect the pedestrian to prove which of the defendants caused the harm B) No, because of the doctrine of alternative liability C) Yes, because a plaintiff always has the burden to prove that a particular defendant's conduct was the factual cause of the plaintiff's physical harm D) Yes, because the pedestrian has failed to offer evidence that the defendants jointly engaged in tortious conduct

(D) is correct. Based on the facts, the pedestrian's action here is likely based on negligence. In a negligence action, the plaintiff has the burden of proving that the defendant breached a duty owed to the plaintiff and that the breach of duty caused the plaintiff's injury. If multiple defendants are acting in concert or jointly engaging in tortious conduct, all of the defendants can be held jointly and severally liable for the injury. Here, the pedestrian has offered no evidence which of the particular defendants threw the chair out the window and has offered no evidence that the defendants were jointly engaged in tortious conduct. Because the pedestrian has failed to establish a prima facie case for negligence, the court should grant the defendants' motion for a directed verdict. (A) is incorrect. In situations where the fact that a particular injury occurred may tend to establish a breach of the duty owed, the plaintiff may be able to rely on res ipsa loquitur to create an inference of liability. However, the plaintiff generally must establish evidence connecting a particular defendant to the negligent conduct to rely on this doctrine. Unless the defendants were jointly engaged in the tortious conduct (which is not the case here), res ipsa loquitur generally may not be used to establish a prima facie case of negligence against any individual party. (B) is incorrect. Under the doctrine of alternative liability or alternative causes, which applies when two or more persons have been negligent but uncertainty exists as to which one caused the plaintiff's injury, once the plaintiff proves that harm has been caused to him by one of the defendants, the burden of proof shifts to defendants, and each must show that his negligence is not the actual cause of the harm. Here, the pedestrian has not shown that all of the defendants were negligent, so this doctrine is inapplicable. (C) is incorrect. The language of this choice is too broad (indicated by the word "always" in the choice). In alternative liability situations the plaintiff will not have the burden to prove that a particular defendant was the factual cause of the harm. However, the facts here do not present such a situation.

A state generally provides funding for the medical care of its residents who cannot afford such care. State law, however, prohibits use of this state funding for surgery for any person who has resided in the state for less than one year, except in emergency situations. A woman moved to the state two months ago seeking permanent employment. Her physician recommends non-emergency surgery to treat a medical condition. The surgery would qualify for state funding if the woman had resided in the state for a year. The woman has sued to invalidate the state law that prohibits state funding of her surgery. Should the woman prevail in her action? A) No, because the law reasonably conserve the state's limited resources B) No, because the law reasonably prevents the expenditure of state funds on transient nonresidents C) Yes, because the law burdens the woman's fundamental right to health care D) Yes, because the law burdens the woman's fundamental right to travel

(D) is correct. The right to travel is protected by the Fourteenth Amendment, and it includes the right of newly arrived citizens to enjoy the same privileges as are enjoyed by other citizens of the state. The law here gives a benefit to long-term residents (people who have been residents more than a year) and so infringes on the right to travel. (A) is incorrect. Because the right to travel is a fundamental right, it cannot be interfered with merely because it is reasonable to do so. Likely, the state would have to show that the interference is necessary to achieve a compelling interest. (B) is incorrect for the same reason that (A) is incorrect. (C) comes to the correct conclusion but for the wrong reason. There is no fundamental right to healthcare for women or men.

A ceramics studio contracted with an artist to produce cups and saucers designed by the artist. The artist was an established designer of collectible ceramic dinnerware, and the studio did production work for many artists who created ceramic dinnerware. The price and quantity term of the contract read: "2,000 sets of the cups and saucers at $5 each, payable on delivery." The contract contained a merger clause. The studio produced the cups and saucers and delivered them along with a bill for $20,000 (4,000 pieces at $5 per piece). The artist refused to pay more than $10,000 (2,000 sets at $5 per set). At the trial of the studio's action against the artist for breach of contract, the studio introduced evidence of an established practice in the studio industry to price cup-and-saucer sets on a per-piece, not a per-set, basis. Is the studio's evidence admissible? A) No, because such evidence would vary an unambiguous term of the contract B) No, because the agreement was completely integrated C) Yes, because evidence of trade usage is always admissible D) Yes, because the usage of trade is offered to give meaning to the contract

(D) is correct. This is a contract for the sale of goods (cups and saucers), so Article 2 of the UCC applies. The UCC provides that a written contract's terms may be explained or supplemented by evidence of course of performance, course of dealing, and usage of trade-regardless of whether the writing appears to be ambiguous. Thus, the studio's evidence will be admissible to give meaning to the terms of the contract. (A) is incorrect. As explained above, Article 2 allows terms to be explained or supplemented even if they do not appear to be ambiguous. (B) is incorrect. Even if the agreement is completely integrated under the parol evidence rule, evidence is admissible to define (as opposed to vary) the terms of the contract. (C) is incorrect. Evidence of trade usage is not always admissible (for example, it could not be used to vary the express terms of a fully integrated, final contract), but it can be used to explain or supplement terms.

While visiting at his son's home, a grandfather tripped on a toy left on the floor by his four-year-old grandson. The grandfather fell and was severely injured. The grandfather regularly visited his son's home and was aware that the grandson routinely left toys scattered about the house. The son had never warned the grandfather to look out for toys. The grandfather brought an action against his son to recover for his injuries. At trial, after the close of evidence, both the grandfather and the son have moved for judgment as a matter of law as to liability. The jurisdiction has abolished intra-family immunity and applies the traditional rules of landowner liability. What action should the court take? A) Deny both motions and submit the case to the jury based on negligence B) Deny both motions and submit the case to the jury based on strict liability C) Grant the grandfather's motion, because the son is liable as a matter of law for failing to warn about the risk of toys being left on the floor D) Grant the son's motion, because the son had no duty to warn that the grandson might leave toys on the floor

(D) is correct. Under the traditional rules of landowner liability, the nature of a duty owed by an owner or occupier of land to those on the premises for dangerous conditions on the land depends on the legal status of the plaintiff with regard to the property, specifically, trespasser, licensee, or invitee. A licensee is one who enters on the land with the landowner's permission, express or implied, for her own purpose or business rather than for the landowner's benefit; social guests are considered licensees. Because the jurisdiction has abolished intra-family immunity and applies traditional rules of landowner liability, the grandfather will have the status of a licensee. The owner or occupier owes a licensee a duty to warn of or make safe a dangerous condition known to the owner or occupier that creates an unreasonable risk of harm to the licensee and that the licensee is unlikely to discover. Here, the facts state that the grandfather regularly visited his son's home and was aware that the grandson routinely left toys scattered about the house. Under these circumstances, the son had no duty to warn the grandfather about the toys and thus has not breached a duty to him. Accordingly, the son's motion should be granted. (A) is incorrect. As discussed above, the son owed no duty to the grandfather to warn him of the toys because the grandfather was aware of them. Hence, the case will not be submitted to the jury because the grandfather has not established a prima facie case of negligence. (B) is incorrect. Strict liability is not applicable here because no animals, abnormally dangerous activities, or defective products are involved. The action here would be based on negligence. (C) is incorrect. As discussed above, the son is not liable for failing to warn about the toys on the floor because the grandfather was aware that the grandson regularly left toys on the floor. Because the grandfather is a licensee, the son owed him no duty to warn.

Police officers were executing a search warrant at a home suspected of containing evidence of illegal gambling. No one was at home when the police arrived. After searching the first floor, the officers went upstairs. A friend of the owner then entered the house carrying a briefcase. He set the briefcase on the floor, opened it, and then heard the officers. He became frightened, left the briefcase sitting in the middle of the floor, and hid in a closet. The police officers returned to the first floor and immediately spotted the briefcase, which they knew was not there earlier. Because the briefcase was open, the officers saw its contents-betting slips-and seized them. Because they knew that someone had entered the house since they arrived, they re-searched the first floor. They found the friend and informed him that he was under arrest, clapped handcuffs on him, and read him his Miranda warnings. One of the officers patted the friend down to check for weapons. The officer noticed a bulge in the friend's pocket. Although the officer knew that the bulge was unlikely to be a weapon, he reached into the pocket anyway, and discovered a package that appeared to be (and later proved to be) heroin. The friend was charged with possession of narcotics. At a suppression hearing, will the court agree with the public defender's contention that the friend's arrest was unlawful? A) Yes, because the police officer who searched the friend knew that he did not have a weapon in his pocket B) Yes, because the friend's mere presence in the house did not give the police probable cause to believe he had committed a crime, and they had no basis for searching him at all, because he did not act toward them in a threatening manner C) No, because the contents of the briefcase gave the police probable cause to arrest the man D) No, because the police had a right to search the friend for gambling slips, and the discovery of the heroin was merely incidental to a lawful search

A police officer may conduct a warrantless search of a person incident to the person's lawful arrest. Any arrest is sufficient, as long as it was lawful (e.g., reasonable and based on probable cause). The contents of the briefcase supplied probable cause to believe that the friend was involved in the gambling operation, and thus his arrest was constitutional. The police had searched the house for gambling paraphernalia pursuant to a search warrant. Upon seeing such paraphernalia in the briefcase, which was not previously present, they had reasonable grounds to believe that the person who left the briefcase was involved in the gambling operation. When the officers found the friend, who had not been present during the initial search, they had reasonable grounds to believe that he had left the briefcase and was therefore involved in the commission of gambling offenses. Thus, the friend's arrest was constitutional/lawful. (A) focuses on the propriety of the search that uncovered the heroin, rather than on the validity of the arrest itself. Do not be sidetracked. The call of the question concerns the lawfulness of the arrest. As has been explained above, the friend's arrest was constitutional and based on grounds entirely independent of the legality of the subsequent search and seizure. Moreover, because the arrest was lawful, it does not matter whether the officer thought the bulge was a weapon. This alludes to whether a valid warrantless frisk was performed. A police officer may frisk a person for weapons without a warrant if the officer has reason to believe the suspect is armed and dangerous. But here, the friend had been placed under arrest. Incident to arrest, a person may be thoroughly searched for weapons or any type of evidence. Thus, (A) is incorrect. Although it is true that, as (B) states, the friend's mere presence in the house did not give probable cause to believe he had committed a crime, his arrest was not based on his mere presence. As has been explained, reasonable grounds to believe that the friend was part of the gambling operation arose from the presence of betting slips in the briefcase and the great likelihood that the friend was the person who brought the briefcase into the house. Thus, (B) incorrectly states the basis for the friend's arrest. Also, (B) incorrectly states that there was no basis to search the friend, because he had not behaved threateningly. In fact, the police may conduct a search incident to a constitutional arrest without actually fearing for their safety. Note also that this second part of (B), similarly to (A), incorrectly focuses on the search of the friend, rather than on the arrest itself. (D) incorrectly asserts a right to search the man independent of any probable cause to arrest him. A search warrant does not authorize the police to search persons found on the premises who are not named in the warrant. However, if the police have probable cause to arrest a person discovered on the premises, they may search him incident to the arrest. Consequently, any right that the police had to search the friend arose from their arrest of him, which was based on probable cause. (D) ignores the necessity of probable cause to arrest. Of course, (D) also attempts the same distraction as (A) and (B); i.e., it focuses on the search of the friend as a means of either justifying or attacking the arrest, rather than on the grounds for the arrest itself.

During the trial of a personal injury case, the plaintiff calls a witness to testify that he saw the defendant spill a slippery substance in the roadway. Following the testimony of the witness, the defendant calls the witness's neighbor, who testifies that the witness has a poor reputation for truthfulness in the community. The plaintiff's attorney then cross-examines the neighbor, asking her, in good faith, if she committed the crime of false pretenses last year. Last year, the neighbor had in fact been charged with and convicted of the crime of false pretenses. The defendant's attorney objects to this question. Should the objection be sustained? A) No, because the neighbor was convicted of the crime of false pretenses B) No, because the plaintiff's attorney asked the question in good faith C) Yes, because the impeaching witness cannot be impeached on collateral matters D) Yes, because such an inquiry is not proper on cross-examination

B. Specific bad acts/general: on CX, just no extrinsic for specific bad acts

Ten years ago, a couple bought a building and moved into its second-floor apartment with their teenage daughter. The couple operated a shoe store on the first floor of the building for many years. When the couple purchased the building, the area was predominantly rural and was zoned for nonresidential use. The municipality's zoning is cumulative. Five years ago, the municipality rezoned the area to single-family residential use. The daughter was not aware of this change, since she was away at college. Recently, the daughter inherited the building from her parents. The daughter immediately moved into the apartment and took over the operation of the shoe store on the first floor. The daughter has learned that a developer is planning to build a large residential community in the area surrounding her building. The daughter has asked her lawyer for advice regarding her ability to continue operating the shoe store. Should the lawyer advise the daughter that she can continue to operate her shoe store? A) No, because the nonconforming use of the building terminated when the daughter's parents died B) No, but the municipality must pay her reasonable compensation for her loss resulting from the change in zoning C) Yes, because the shoe store is a nonconforming use D) Yes, because the zoning is cumulative and the building is also used for single-family residential purposes

C) is correct. A cumulative zoning ordinance creates a hierarchy of uses of land, and land that is zoned for a particular use may be used for the stated purpose or for any higher use. A residential use is higher than a nonresidential use. Here, the building was in an area originally zoned for nonresidential use. The daughter and her parents used the property for a business and their residence. This was appropriate under the cumulative zoning ordinance as the family's uses met or exceeded the zoned use. Later, the area was rezoned for single-family residential use, which is a higher use than the shoe store. However, a use that exists at the time of passage of a zoning ordinance and that does not conform cannot be eliminated at once. Generally, the nonconforming use may continue indefinitely, but any change in the use must comply with the zoning ordinance. Because the shoe store existed at the time of the rezoning, the daughter may continue to operate the shoe store as a nonconforming use. (A) is incorrect. Generally, the nonconforming use may continue indefinitely. The key is the use of the property and not the ownership. The parents' deaths do not affect the zoning status of the building. Thus, the daughter may live in the building and operate the shoe store. (B) is incorrect. The zoning power is limited by the "no taking without just compensation" clause of the Fifth Amendment. Nevertheless, zoning is generally not a "taking" unless it amounts to a physical appropriation of the property or denies the owner of all economic use, which would not be the case here (the daughter could use the property as a single-family residence). Because the property is a nonconforming use, she will be allowed to continue that use. (D) is incorrect. A cumulative zoning ordinance creates a hierarchy of uses of land, and land that is zoned for a particular use may be used for the stated purpose or for any higher use. A single-family home is a higher use than an apartment building or a commercial use such as a shoe store. Because the shoe store is a lower use than a single-family residence, it would not be allowed in this area. Nor would the daughter's apartment above the store. The store and apartment are allowed only because they are a nonconforming use.

A mechanic sued his former employer in federal court, claiming that the employer had discharged him because of his age in violation of federal law. The employer answered, denying the claims and promptly moving for summary judgment. In support of the motion, the employer attached the mechanic's employment evaluations for the past three years, which rated his skills and performance as poor and culminated in a recommendation for his discharge. What is the mechanic's best argument to defeat the summary judgment motion? A) The allegations in the complaint conflict with the mechanic's employment evaluations, raising a genuine dispute as to material facts B) The employer cannot rely in his motion on matters outside the pleadings C) The essential facts are unavailable to the mechanic and therefore discovery is required D) The motion was flied before the close of discovery

C) is correct. If the mechanic (the nonmovant) shows by affidavit or declaration that he cannot present facts essential to justify his opposition to the summary judgment motion, Rule 56(d) authorizes him to ask the court to defer action or deny the motion to allow time to obtain affidavits or declarations or to take discovery. The employer moved for summary judgment right after answering and before any discovery. That timing would support defeating the summary judgment motion at this time. (A) is incorrect. Under Rule 56(c), a party asserting that a fact is genuinely disputed must support the assertion by citing particular parts of the record, including affidavits or declarations, stipulations, or discovery materials. The mechanic cannot simply rely on the complaint allegations to rebut the employer's evidence but must support his factual position with his own evidence that a factual dispute exists. If he cannot do so, Rule 56(d) authorizes him to ask the court to defer action or deny the summary judgment motion to allow time to obtain affidavits or declarations or to take discovery. (B) is incorrect. The function of a summary judgment motion is to allow additional evidence outside the pleadings to show that there is no genuine dispute of fact and that the movant is entitled to judgment as a matter of law. Rule 56(c)(1) enumerates the types of materials that the moving party may use to support a summary judgment motion, including documents. If the mechanic (the nonmovant) shows by affidavit or declaration that he cannot present facts essential to justify his opposition to the summary judgment motion, Rule 56(d) authorizes him to ask the court to defer action or deny the motion to allow time to obtain affidavits or declarations or to take discovery. (D) is incorrect. The fact that a summary judgment motion is filed before the close of discovery does not require the court to deny it. Under Rule 56(b), a party may file the motion at any time until 30 days after the close of all discovery. The problem here is that the employer filed the motion before discovery commenced, thus providing the mechanic an argument to defer action or deny the motion under Rule 56(d)

An accountant employed by the Federal Communications Commission was offended by various jokes and cartoons that employees would post in the office cafeteria. The Commission did not have any rules regarding what employees could post in the cafeteria, and none of the cartoons were pornographic or harassing. Nevertheless, the accountant lodged a number of complaints with his supervisor that went unheeded. Finally, the accountant posted his own notice chastising the hypocrisy and immorality of the agency for allowing such cartoons when it was charged with ensuring a standard of decency on the public airwaves. The notice prompted a great deal of debate among employees and a great deal of displeasure on the part of the accountant's supervisor, particularly after it was posted on another employee's blog and received some media attention. A labor contract between the agency and the clerical workers' union contained a policy for providing for termination of union employees only for certain specified grounds, but the accountant was not a member of the union and was not covered by the policy or any other employment agreement. Which of the following statements is most accurate regarding the agency's right to dismiss the accountant? A) The accountant has a liberty interest in the exercise of his First Amendment rights that entitles him to a hearing to contest the grounds of his dismissal B) The accountant has a property interest as a public employee that precludes him from being fired without notice and an opportunity to respond C) The accountant has no right to a hearing because his statements were not an expression of view on public issues D) The accountant has both a liberty interest and a property interest that entitles him to a pretermination evidentiary hearing

The accountant is entitled to a hearing because he has a liberty interest in the exercise of his First Amendment rights. If the accountant is fired, he has a right to a hearing to determine whether his First Amendment rights were violated by his dismissal. Under the Due Process Clause of the Fifth Amendment, a person has a liberty interest in the exercise of specific rights provided by the Constitution, including freedom of speech. A government employee may not be fired for expressing his views regarding public issues, but can be fired for speech that disrupts the employer's policies or undermines the employer's authority. Under the Court's expansive interpretation of what a public issue is in this context [see Rankin v. McPherson (1987)], the accountant's statement would probably qualify. At the very least, he can make enough of a showing that his termination violates his free speech rights to be entitled to a hearing on the issue under procedural due process principles. [See Givhan v. Western Line Consolidated School District (1979)] (B) is wrong because the accountant does not appear to have a property interest in his job. A public employee who is subject to removal only for "cause" has a property interest in his job and must be given notice of the charges against him that are to be the basis for his job termination, and a pretermination opportunity to respond to those charges. Here, however, the accountant did not have a property interest in his job. He could have been dismissed for no reason at all. He was not covered by the labor contract between the agency and its clerical workers, and there appears to be no other basis for him to claim an entitlement to continued employment. (C) is wrong because the accountant is entitled to a hearing as long as he can raise a prima facie claim that his speech, which was regarding an issue important to the perception of his agency, was on a public issue and therefore protected by the First Amendment. (D) is wrong for two reasons: As discussed above, the accountant does not have a property interest in his job. Also, due process does not necessarily entitle him to a pretermination evidentiary hearing; a post-termination evidentiary hearing is probably sufficient. [See Cleveland Board of Education v. Loudermill (1985)]

An employee filed in state court a civil action alleging sexual harassment in the workplace. She asserted federal statutory employment discrimination claims against her employer, and she asserted a state law battery claim against the co-worker who allegedly engaged in the sexual harassment. The plaintiff employee, the defendant employer, and the defendant co-worker are all citizens of the state in which the action was filed. May the defendants properly remove the action to federal district court? A) No, because the federal district court does not have subject matter jurisdiction over the action B) No, because the defendants are citizens of the state in whose court the action is pending C) Yes, the action may be removed to federal district court, provided that amount in controversy exceeds 75K D) Yes, the action may be properly removed to federal district court, regardless of the amount in controversy

The action may be properly removed regardless of the amount in controversy. A defendant may remove an action that could have originally been brought in the federal courts. Federal question jurisdiction is available when the plaintiff, in her well-pleaded complaint, alleges a claim that arises under federal law. In the instant case, the plaintiff is bringing a federal statutory employment discrimination claim. This presents a federal question, and removal is authorized on this basis. Once one claim satisfies the requirements for original federal subject matter jurisdiction, the court has discretion to exercise supplemental jurisdiction over related claims that derive from the same common nucleus of fact and are such that a plaintiff would ordinarily be expected to try them in a single judicial proceeding. The battery claim arises out of the same transaction or occurrence as the employment discrimination claim. Thus, the court has supplemental jurisdiction over that claim, making (D) correct and (A) incorrect. (B) is incorrect because removal would be based on federal question jurisdiction, and the restriction on an in-state defendant removing a case to federal court applies only when removal is based on diversity of citizenship jurisdiction. Similarly, federal question jurisdiction, unlike diversity of citizenship jurisdiction, does not have an amount in controversy requirement, making (C) incorrect.

An investor who owned several thriving shopping malls was negotiating to purchase a local mall from the company that currently owned it. A staff attorney for the state transportation department who shopped at the mall regularly learned of the negotiations and contacted the investor. The mall had deteriorated noticeably during the time the current company had owned it and the attorney believed that new ownership would revitalize the mall considerably. Although the attorney had no information to support this, she told the investor that the state was currently planning to construct a new interchange for the turnpike only three blocks from the mall. The investor went ahead with the purchase, believing that the new interchange would boost sales. In fact, no interchange was being considered by the state at that time, and nothing that the investor did after he purchased the mall could stem the decline in sales. He ended up selling the property at a substantial loss several years after the purchase. Does the investor have a cause of action against the attorney for his losses? A) Yes, for the negligent misrepresentation, because the owner made a business transaction in reliance on the attorney's statement B) Yes, for intentional misrepresentation, because the attorney was aware that she did not know whether the state was planning an interchange C) No, because the attorney's statement pertained to a future event that may not be justifiably relied upon D) No, because the attorney made her statement to the owner gratuitously

The attorney acted with scienter for purposes of an intentional misrepresentation action because she was aware that she did not know whether the state was planning an interchange. To establish a prima facie case of intentional misrepresentation or fraud, plaintiff must prove (i) misrepresentation by defendant, (ii) scienter, (iii) intent to induce plaintiff's reliance on the misrepresentation, (iv) causation (actual reliance on the misrepresentation), (v) justifiable reliance on the misrepresentation, and (vi) damages. The element of scienter, which involves defendant's state of mind, requires plaintiff to show that defendant made the statement knowing it to be false or made it with reckless disregard as to its truth or falsity. Because the attorney made her statement even though she had no information that the state was planning an interchange, she acted with scienter. The other elements of intentional misrepresentation are established by the facts. Thus, the investor has a cause of action against the attorney under the condition stated in (B). (A) is incorrect because an action for negligent misrepresentation is not supported by these facts. Negligent misrepresentation requires (i) a misrepresentation made by defendant in a business or professional capacity, (ii) breach of duty toward that particular plaintiff, (iii) causation, (iv) justifiable reliance, and (v) damages. Here, even though the investor was involved in a business transaction, the attorney was not. She was not acting in a business capacity but rather for her own personal interests. Hence, she is not liable for negligent misrepresentation. (C) is incorrect because the attorney's statement was a false representation of an existing fact-that an interchange was currently being planned. If the attorney had instead assured the owner simply that the interchange was going to be built in the future without any assertion of present facts, the investor could not justifiably rely on the statement because it is a statement of a future event over which the attorney did not have control. The statement here is actionable because an interchange was not even being planned. (D) is incorrect. The fact that the attorney made the statement to the owner gratuitously rather than in a commercial transaction absolves her from liability for negligent misrepresentation, but it has no relevance to her liability for intentional misrepresentation.

A three-year-old girl attending nursery school punched a boy in the face because he was teasing her about wearing glasses. The blow knocked out the boy's newly acquired front teeth. If the boy's parents sue the girl's parents for the injury, what is their best defense? A) The boy was the initial aggressor B) The girl is too young to be responsible for her actions C) A parent cannot be liable for damages due to the child's conduct D) The parents were unaware of any potentially violent behavior by the girl

The best defense of the girl's parents is that they were unaware of any potentially violent behavior by the girl. At common law, parents are not vicariously liable for the torts of their child. (Statutes in most states allow for limited liability for intentional torts, but there is no indication of such a statute here.) Parents can be liable, however, for their own negligence, i.e., in not exercising due care under the circumstances. Thus, if the parents know their child may be violent, they could be negligent if they do not take precautions to prevent that behavior or injury from that behavior. However, if the parents have no reason to know their child could be violent, they have no duty to protect against such behavior. Here, if the girl had never done anything like this before, and her parents had no idea that she would be violent, they were not negligent in allowing her to attend nursery school. (A) is wrong because, although the boy's teasing may have provoked the girl, he did not initiate the violence. He did nothing to allow the girl a right of self-defense, and so his actions would not provide the girl's parents with a good defense. (B) is wrong because there is no general tort immunity for children. As long as the child is old enough to intend the act, she can be held liable. Here it seems that the girl intended to cause a battery. She either intended or knew with substantial certainty that swinging her fist would strike the boy in the face, i.e., would cause a harmful or offensive contact. Thus, this choice does not present the best defense for the defendants. (C) is wrong because parents can be liable for damages due to their child's conduct. As explained above, although the parents are not vicariously liable at common law, they can be liable based on their own negligence (e.g., for negligent supervision).

An owner and a builder executed a contract providing that the builder was to construct a residence on a specified lot according to plans and specifications. The total contract price was $800,000. No date was included in the contract for completion of the home. After the builder completed 60% of the residence, a flash flood from a nearby river partially eroded the lot but left the construction undamaged. The builder determined that it would cost an additional $1.7 million to repair the lot so that the residence can be constructed according to the plans. Without the additional lot repair work, the residence cannot be constructed at all. Which of the following states the probable legal consequences of the lot erosion? A) The builder may avoid the contract if the increased costs of construction would bankrupt him B) The contract is void because of mutual mistake C) The builder is discharged from his duties under the contract because of impracticability D) The builder remains obligated to perform under the contract, but he may bring an action against the owner for the increased costs of construction

The builder will be discharged from his duties under the contract. Modern courts recognize that impracticability due to excessive and unreasonable difficulty or expense is a defense to breach of contract for nonperformance. Since the cost to the builder to perform under the original contract would exceed more than double what he would be paid, he likely would be excused from performance by commercial impracticability. (D) is therefore incorrect. Unlike destruction of the building itself before completion, which will not discharge a contractor's duty, the erosion of the lot, which destroys the means of performing the contract, will generally not be one of the risks that a builder will be deemed to have assumed. (A) is wrong because the builder's bankruptcy is irrelevant for this purpose. (B) is wrong because there is no mutual mistake here

A minor entered a liquor store and asked the cashier to let him purchase a case of beer for a party even though he was underage. The cashier agreed as long as he paid double the retail price for the beer as compensation for the risks of the sale. As the cashier started to ring up the transaction, an undercover officer who overheard them intervened and arrested them. The state criminal code provides that it is a felony to knowingly provide alcohol to any person under the age of 21. If the state follows the unilateral theory of conspiracy, of what crimes can the minor and cashier be convicted? A) The minor can be convicted of solicitation and the cashier can be convicted of conspiracy to violate the statute B) Both the minor and the cashier can be convicted of conspiracy to violate the statute C) The minor cannot be convicted of either solicitation or conspiracy, but the cashier can be convicted of conspiracy to violate the statute D) The minor cannot be convicted of either solicitation or conspiracy, and the cashier cannot be convicted of conspiracy

The cashier can be convicted of conspiracy in a unilateral jurisdiction even though the minor can be convicted of neither solicitation nor conspiracy. Under the unilateral approach adopted by the Model Penal Code, the crime of conspiracy requires that only one party have genuine criminal intent, and may be shown by proof that the defendant agreed with another to commit a crime, even if the other person does not share the commitment. Thus, the fact that no other party to the conspiracy could be found guilty does not prevent the defendant from being convicted of conspiracy. Here, the fact that the minor is a member of the class that the statute was designed to protect prevents him from being found guilty of conspiracy. (B) is therefore incorrect. However, this fact has no bearing on the cashier's liability for conspiracy under the unilateral approach. Thus, (C) is correct and (D) is incorrect. (A) is incorrect because the crime of solicitation is treated the same as conspiracy. If the solicitor could not be guilty of the completed crime because of legislative intent to exempt him, he cannot be guilty of solicitation of the crime. Because the minor is a member of the class intended to be protected by the statute, he cannot be found guilty of soliciting the cashier to complete it.

A plaintiff sued an auto manufacturer for negligence after a car accident involving the plaintiff's car that was made by the auto manufacturer. Sixty days after service of the complaint and 40 days after service of the manufacturer's answer that contained no counterclaim, the plaintiff filed a motion seeking to file an amended complaint adding a claim for strict products liability against the auto manufacturer stemming from the same incident. The statute of limitations for strict products liability claims expired one week before the motion was filed. How should the court rule on the plaintiff's motion? A) Grant the motion, because every party is entitled to amend once as a matter of course B) Grant the motion, because the amended complaint relates back C) Deny the motion, because, while the motion is timely, the proposed claim is futile because the statute of limitations has run D) Deny the motion, because it is not timely

The court should grant the motion. Federal Rule of Civil Procedure 15 states that leave of court (to grant motions to amend) is to be "freely given when justice so requires." The rule does not provide any clear date when amendments are no longer permissible, although later amendments obviously would be less fair and less likely to be considered in the interest of justice. Additionally, for statute of limitations purposes, proposed claims may be considered to "relate back" to the date of the original pleading in which the claim was made under Rule 15(c). That is critical here because, although the statute of limitations for the proposed products liability claim had expired at the time the motion was made, the amended complaint asserting a products liability claim relates back to the original filing because that claim stems from the same facts alleged in the original complaint, and the statute of limitations had not expired at the time of the original filing. (A) is incorrect because a plaintiff may amend the complaint once as a matter of course (i.e., without court intervention) not later than 21 days after service of the auto manufacturer's answer. Here, 40 days have passed since the auto manufacturer filed its answer, so amending as a matter of course is not permitted. (C) is incorrect. Although the proposed claim would be futile if filed in an original complaint, here, because the claim can relate back to a complaint filed at a time when the proposed claim was still viable, the proposed claim is not considered futile. (D) is incorrect by suggesting that the motion is not timely. Although plaintiff can no longer amend as a matter of course, there is no absolute date pursuant to which a motion to amend is untimely.

A three-car accident occurred in a city in the Northern District of State A. The cars were driven by a citizen of State B who resides in the Southern District of State B, a citizen of State A who resides in the Southern District of State A, and a citizen of State C who resides in the Northern District of State C. The State B citizen filed a negligence action against the other two drivers in the United States District Court for the Southern District of State A. Although the two defendants believed that venue was improper, neither filed a pre-answer motion objecting to venue. They instead proceeded to file their answers, responding to the merits of the claim. The State C defendant, however, included in her answer a motion to dismiss the action for improper venue. How should the court rule on the State C defendant's motion to dismiss for improper venue? A) Deny the motion, because the defendants waived the defense of venue by not asserting it in a separate, pre-answer motion prior to filing their answer addressing the merits of the action B) Deny the motion, because all the defendants must object to venue before the court may dismiss on venue grounds C) Deny the motion, because venue is proper D) Grant the motion, because the motion is timely and venue is proper

The court should grant the motion. Objections to venue may be raised in a defendant's answer if the defendant did not assert a Rule 12(b) pre-answer motion, as is the case here. Venue here is improper because (i) none of the events giving rise to the claim occurred in the Southern District of State A, and (ii) all defendants do not reside in State A. (A) is incorrect because a defendant can object to venue in a pre-answer motion or in the answer. (B) is incorrect because all defendants do not have to object to venue before the court may dismiss the case on venue grounds. (C) is incorrect because venue is improper, as explained above.

A builder from State A sued a homeowner from State B for breach of contract in federal court, alleging that the homeowner failed to pay the second half of the agreed-upon price for completion of construction on a house. The process server attempted to serve the summons and complaint on the homeowner at the house, but after two failed attempts, instead served it on an elderly next-door neighbor, who was out in the front yard and volunteered to accept it on the homeowner's behalf. The homeowner then filed a motion to dismiss for insufficient process. Assume that the requirements for service of process in both states are identical to the requirements of the Federal Rules of Civil Procedure. Is the court likely to dismiss the action for insufficient process? A) Yes, because the homeowner did not designate the neighbor as an authorized agent to receive service of process B) Yes, because the neighbor is not considered one residing at the homeowner's place of abode, even though the neighbor is of suitable age C) No, because the homeowner filed the wrong motion D) No, because the homeowner has no meaningful defense for insufficient service of process since she had actual notice

The court will not likely dismiss the action. Insufficient process refers to defects in the documents and their content, whereas insufficient service of process refers to the manner in which the documents were presented, delivered and/or served to the defendant. The key to this answer is reading comprehension. The facts state that the homeowner filed a "motion to dismiss for insufficient process," not "insufficient service of process," which are two separate grounds for dismissal. [See Fed. R. Civ. P. 12(b)(4) and (5), respectively] (A), (B) and (D) are all wrong because they all relate to issues regarding insufficient service of process, which is not relevant to the call of the question. (C) is correct, because the homeowner filed the wrong motion. The homeowner should have filed a motion to dismiss based on insufficient service of process. Generally speaking, service of process is sufficient if it is made by: (i) personal service, (ii) service left at the defendant's usual place of abode with one of suitable age and discretion, or (iii) service upon an authorized agent of the defendant. Alternatively, service may be made under state rules or by mail under the waiver of service provision [see Fed. R. Civ. P. 4(d)].

An acquaintance asked the defendant to give him a lift downtown because he did not have bus fare. While riding on the defendant's motorcycle, the acquaintance asked to stop at a convenience store to get a bottle of wine, showing the defendant a tire iron in his backpack that he was going to use. The defendant nodded in acknowledgment of what the acquaintance was planning to do. The defendant stopped at the store and waited in the parking lot while the acquaintance went in. He demanded money from the clerk, brandishing the tire iron. The clerk tried to grab a gun under the counter while he was filling a bag with money, and a struggle ensued. The gun discharged, killing the clerk. The defendant heard the gunshot and raced off, but was eventually apprehended. The jurisdiction's criminal code provides that a death caused during the commission of certain felonies, including robbery, is first degree felony murder, for which the death penalty is permitted. The code also permits cumulative penalties for first degree felony murder and for the underlying felony. The defendant was charged and convicted of both robbery and felony murder. After appropriate consideration of all relevant circumstances, the jury imposed the death penalty. On appeal, the defendant challenged both the convictions and the sentence. Assuming that the above facts were properly admitted into evidence, how should the appellate court rule? A) The defendant's conviction for both offenses should be upheld, but imposition of the death penalty was not proper B) The defendant's conviction for both offenses should be upheld, and imposition of the death penalty was proper C) The defendant's conviction should be overturned under double jeopardy principles because robbery is a lesser included offense of felony murder D) The defendant's conviction for felony murder should be overturned because the circumstances do not establish the necessary degree of culpability

The defendant can be found guilty of robbery and felony murder, but the death penalty cannot be imposed. The defendant can be found guilty of robbery as an accomplice. The Supreme Court has held that, under the Eighth Amendment, the death penalty may not be imposed for felony murder where the defendant, as an accomplice, did not take or attempt or intend to take life, or intend that lethal force be employed. [Enmund v. Florida (1982)] Here, because the defendant's involvement in the crime was only to provide transportation, it cannot be said that he participated in such a major way that he acted with reckless indifference to human life; hence, the death penalty cannot constitutionally be imposed against him. (B) is therefore incorrect. (C) is incorrect because the defendant's conviction of both robbery and felony murder does not raise double jeopardy problems under these facts. Under the rule that lesser included offenses "merge" into greater offenses, a person may not be convicted of both the greater offense and a lesser included offense. While the Supreme Court has held that a subsequent prosecution for robbery is not permitted against a defendant who has been tried for felony murder where the robbery is the underlying felony, this situation is different. Imposition of cumulative punishments for two statutorily defined offenses arising from the same transaction and constituting the same crime does not violate double jeopardy when the punishments are imposed at a single trial, as long as the two offenses were specifically intended by the legislature to carry separate punishments. [Missouri v. Hunter (1983)] Here, the legislature did specifically provide for cumulative penalties for first degree felony murder and for the underlying felony. Thus, the defendant can be convicted of both robbery and felony murder. (D) is incorrect because the jury could properly find the defendant guilty of felony murder. When the felony murder rule is combined with accomplice liability rules, the scope of liability becomes very broad. The felony murder rule provides that a killing-even an accidental one-committed during the course of a felony is murder. All parties to the felony are liable for the murder as long as (i) it was committed during the commission of the felony or in fleeing from the scene, and (ii) it was a foreseeable result of commission of the felony. Courts have been willing to find most deaths committed during a felony to be foreseeable. Here, the jury could reasonably find the shooting death of a store clerk by the acquaintance during a struggle for a gun to be a foreseeable result of the commission of a robbery and impose felony murder liability on the defendant as an accomplice because he knew that the acquaintance was going to commit a robbery.

The defendant's neighbor owned an authentic major league baseball signed by Babe Ruth. The defendant asked if he could show it to some friends who were visiting. The neighbor agreed as long as he kept it in the display case, which the defendant promised to do. In fact, the defendant intended to use the ball in a pickup game. During the game, the ball was hit over the fence and into a yard with a guard dog, which had chewed up several other balls that had previously landed in the yard. The dog did the same to that ball. When the neighbor learned what happened to the ball, he pressed charges against the defendant. If the defendant is convicted, he will most likely be found guilty of what crime? response - incorrect A) Common law larceny B) Embezzlement C) False pretenses D) Larceny by trick

The defendant is guilty of larceny by trick because he obtained possession of the baseball by means of a misrepresentation. Larceny is the taking and carrying away of tangible personal property of another by trespass, with intent to permanently (or for an unreasonable time) deprive the person of her interest in the property. The taking must be without the consent of the person in possession of the property. If such consent is induced by a misrepresentation of a past or existing fact, the consent is not valid. The resulting larceny is called larceny by trick. Here, the defendant obtained possession of the baseball with the owner's consent. However, this consent was obtained by means of the defendant's misrepresentation about friends visiting. This was a false statement of an existing fact, made with the intent that his neighbor rely on the statement, and the misrepresentation induced his neighbor's consent. At the time of this taking, the defendant intended to deal with the baseball in a manner that involved a substantial risk of damage or loss. This suffices as intent to permanently deprive. Therefore, all the elements are in place for larceny by trick. (A) is not as good a choice as (D) because the taking in this case is better characterized as larceny by trick rather than larceny, given that the defendant induced his neighbor to consent to his taking possession of the baseball. (C) is incorrect because the defendant obtained only possession of the baseball, not title. False pretenses differs from larceny by trick in what is obtained. If the defendant obtains only possession of the property, the offense is larceny by trick, whereas obtaining of title means that false pretenses has been committed. What the victim intended to convey to the defendant is determinative. The neighbor intended only to let the defendant borrow the baseball for a short time, not to convey title to him. Consequently, the only thing the defendant obtained was possession of the baseball. Because title to the baseball was not obtained, there can be no conviction of false pretenses. Regarding (B), embezzlement is the fraudulent conversion of property of another by a person in lawful possession of that property. In embezzlement, misappropriation occurs while the defendant has lawful possession of the property, while in larceny, it occurs generally at the time the defendant obtains wrongful possession of the property. Here, as detailed above, the defendant's taking of possession of the baseball was trespassory due to the manner in which he obtained consent to such possession. The crime of larceny was complete on the defendant's taking possession with the requisite intent to permanently deprive. Thus, at the time the baseball was destroyed, the defendant had already misappropriated it and was not in lawful possession of it. As a result, there can be no conviction for embezzlement.

An entrepreneur entered into a written lease-option to purchase an office building. The option to purchase the building could be exercised at any time during the five-year term of the lease by giving the building's owner a written 30-day notice of the intent to exercise the option. A few months later, the entrepreneur assigned his lease-option to a dentist looking for office space by written agreement. If the dentist fully performs under the lease, can the dentist exercise the option to purchase that was given to the entrepreneur? A) Yes, because both the burden and the benefit of the covenant to convey run with the land B) No, because the covenant to convey does not touch or concern the land C) No, because the option to purchase was personal to the entrepreneur D) No, because the burden of the covenant to convey given in a lease does not run with the land

The dentist can exercise the option to purchase. A covenant to convey touches and concerns both the leasehold and reversion, and therefore runs with those respective interests in the land. Thus, (B) is incorrect. (C) is incorrect because there is nothing about the option in the facts that shows it to be personal. (D) is incorrect because, as explained above, the burden of the covenant to convey does run with the land.

An entrepreneur opened a specialized business on her land. After using up most of her capital to purchase inventory, however, the entrepreneur needed more funds and asked her friend for a $30,000 loan, to be secured by the business's inventory. The friend declined the loan. A desperate entrepreneur then told the friend she would convey the land, which had a fair market value of $100,000, to him if he would give her the loan at the current market rate of interest. The friend agreed, and the entrepreneur conveyed the land to the friend the next day. At that time, the friend gave the entrepreneur $30,000 in cash, and the parties orally agreed that the entrepreneur would pay the friend back at the rate of $1,000 per month, and that after the loan was paid in full, the friend would reconvey the land to the entrepreneur. The friend immediately recorded his deed to the land. The entrepreneur made three $1,000 payments to the friend and then paid no more. She continued to live on the land but, being very much in debt, could not repay the loan. The friend, meanwhile, had received an offer to buy the land for $100,000. Which of the following most accurately states the friend's right to sell the property? A) The friend may sell the land and keep the entire proceeds B) The friend may sell the land, but he must give 73,000 of the proceeds to the entrepreneur C) The friend may sell the land only after formally foreclosing on the property D) The friend may not sell the land

The friend may sell the land, but only after formally foreclosing on the property. If a deed is given for security purposes rather than as an outright transfer of the property, it will be treated as an "equitable" mortgage and the creditor will be required to foreclose it by judicial action like any other mortgage. In determining whether an absolute deed is really a mortgage, the court considers the following factors: (i) the existence of a debt or promise of payment by the deed's grantor; (ii) the grantee's promise to return the land if the debt is paid; (iii) the fact that the amount advanced to the grantor/debtor was much lower than the value of the property; (iv) the degree of the grantor's financial distress; and (v) the parties' prior negotiations. Here, the entrepreneur owed the friend a debt; the friend promised to return the property if the debt was paid; the amount advanced ($30,000) was much lower than the value of the property ($100,000); the entrepreneur was in great financial distress; and the parties' negotiations reveal that this transaction was intended as security for the loan. Thus, the friend must bring a judicial foreclosure proceeding before he can sell the land. (A) is wrong because a foreclosure is required. Furthermore, even in a foreclosure sale, the friend is not entitled to all of the proceeds. The proceeds are used to first pay the expenses of the sale, attorneys' fees, and court costs; then to pay the principal and accrued interest on the loan that was foreclosed; then to pay off junior interests. Any remaining proceeds are returned to the mortgagor. The friend is entitled only to his expenses and the amount still owing on the $30,000 loan, including accrued interest. Because the friend has a buyer willing to pay $100,000, the entrepreneur should get some money back. (B) is wrong for two reasons: (i) as explained above, the friend cannot sell the property without a judicial foreclosure; and (ii) the entrepreneur would not be entitled to $73,000. The friend is entitled to his expenses of sale and the principal amount owing, plus accrued interest. (D) is wrong because the friend can sell the land, provided he undertakes formal foreclosure proceedings.

A landowner conveyed his land "to my son for life, then to my son's widow for her life, then to my son's children." At the time of the conveyance, the son was 20 years old and unmarried. The son eventually married and had two children, the landowner's grandson and granddaughter. Many years later, the landowner and the grandson were involved in a train accident. The landowner was killed instantly. The grandson died a short time later of his injuries. The landowner left his entire estate by will to his friend. The grandson's will devised his entire estate to the city zoo. The son's wife was so grief-stricken that she became ill and died the next year, leaving her entire estate to her husband. Eventually the son met and married a 21-year-old. Ten years later, the son died, leaving everything to his second wife. When the second wife moved onto the land, the granddaughter filed suit to quiet title to the land, joining all of the appropriate parties. If the jurisdiction recognizes the common law Rule Against Perpetuities, unmodified by statute, in whom will the court most likely find that title to the land is held? A) One-half in the granddaughter and one-half in the city zoo, subject to the second wife's life estate B) One-half in the granddaughter and one-half in the second wife, because the second wife, took the son's interest C) Entirely to the friend, subject to the second wife's life estate because the gift to the son's children violate the Rule Against Perpetuities D) Entirely in the granddaughter, subject to the second wife's life estate, because the grandson did not survive the son

The granddaughter and the city zoo each own one-half of the land, subject to the second wife's life estate. At the time of the conveyance by the landowner, the son had a life estate, the son's widow had a contingent interest (because the son's "widow" cannot be ascertained until the son's death), and the son's children had a contingent remainder (because they have not yet been born). When the grandson and the granddaughter were born, however, their interests became vested subject to open (i.e., if the son had more children). Thus, when the grandson died, he had a vested remainder subject to open that he was free to devise by will; the city zoo took his vested remainder subject to open. At the son's death, the class of his "children" closed (because the son could not have any more children), and the granddaughter's and the zoo's vested remainders subject to open became indefeasibly vested. Also at the son's death, his widow was ascertained and her interest vested in possession. Because the second wife was the son's widow, she is entitled to the valid life estate. Thus, the granddaughter and the city zoo hold one-half interests, subject to the second wife's life estate. (B) is wrong because the son had no interest in the land when he died. He merely had a life estate, which ended at his death. He did not inherit any interest in the property from anyone else. The only person he inherited from in these facts was his first wife, and she had no interest in the land. Furthermore, this choice overlooks the city zoo's interest, which was inherited from the grandson. (C) is wrong because the son's children's interest does not violate the Rule Against Perpetuities. To be valid under the Rule, an interest must vest if at all within a life in being at its creation plus 21 years. The son is a life in being. At the son's death, his children's interest is certain to vest or fail: If the son had any children, at his death, the children's interest would become indefeasibly vested (i.e., the class would close and the children's interest would no longer be subject to open). Note that the children need not come into possession within the perpetuities period; the only requirement is that their interests vest within the period. Likewise, if the son had no children, the gift to them was certain to fail at his death. Thus, the children's interest does not violate the Rule. Because the son had children and their interest was valid, there was no interest to revert to the landowner and to be devised to the friend. Note that the unborn widow aspect of this question is a red herring. The fact would be relevant only if the children's gift were conditioned on their surviving the widow, in which case the takers would remain unascertained and their interest would remain contingent until that time. But because the children's interest vested at the son's death, it is irrelevant that the son's "widow" was not a life in being at the creation of the interest. (D) is wrong because the gift to the son's children was not conditioned on their survival of the son. The law does not imply such a condition. The grandson's interest was vested subject to open and could be disposed of by his will.

While executing a search warrant at the home of a suspected arsonist, the police heard a knock on the door. A plain clothes officer answered the door and found a young woman standing outside with a backpack in her hand. The woman asked for the owner of the home by name and was told that the owner was not available at the moment. The woman replied, "Give him this, and tell him thanks for the $8,000," and she gave the officer the backpack. The officer opened the backpack and found eight hi-tech delay timers. The officer then identified himself as a police officer and placed the woman under arrest. He performed a quick patdown of the woman's outer clothing. In her jacket pocket, the officer found a package that appeared to be (and later proved to be) heroin. In addition to arson-related charges, the woman was charged with possession of heroin. At trial, the woman's attorney moved to have the heroin excluded from evidence. How should the court rule? A) For the woman, because the warrant only entitled the police to search the premises B) For the woman, because the heroin was seized during an unlawful arrest C) For the state, because the heroin was seized during a lawful arrest D) For the state, because searching the backpack was within the scope of the warrant

The heroin was discovered during a search incident to a lawful arrest and is admissible. A search warrant does not authorize the police to search persons found on the premises who are not named in the warrant. However, if the police have probable cause to arrest a person discovered on the premises, they may search him incident to the arrest. This search may be of the person and areas into which he might reach to obtain weapons or destroy evidence (his "wingspan"). The arrest of the woman was lawful because the presence of the timers she brought to the house gave the police probable cause to believe that she was involved in the arsons being investigated. Because the arrest of the woman was lawful, the police were entitled to conduct a search incident to that arrest. Such a search was permissible even though the police did not actually fear for their safety. Consequently, the heroin discovered as a result of this search is admissible, and the motion to suppress will be denied. (A) is incorrect because, although the warrant only authorized a search of the premises, this does not preclude the police from also searching persons found on the premises as to whom there exists probable cause to arrest. Once the woman was lawfully arrested, the police were fully entitled to search her incident to the arrest. (B) is incorrect because, as explained above, the arrest of the woman was lawful. Thus, the heroin cannot be suppressed as the product of an unlawful arrest. (D) is not as good a choice as (C) because whether the backpack was covered by the warrant to search the house is irrelevant to the woman's charge. Once the woman relinquished possession of the backpack by giving it to the officer, she did not have a legitimate expectation of privacy in the object, nor did she have an expectation of privacy in the home (as it was not her home and she was not an overnight guest); hence, she did not have standing to object to the backpack being searched, even if it was not covered by the warrant to search the home.

The owner of a television agreed to sell it to a neighbor for $250. The neighbor made a down payment of $70, took possession of the television and agreed to pay the outstanding balance in nine equal $20 installments, beginning on June 5, with subsequent installments due on the fifth of each month until the balance was paid in full. The neighbor's friend owed her $200. On May 20, the neighbor and her friend entered into an oral agreement whereby the friend agreed to make the nine $20 installment payments to the seller in exchange for the neighbor's promise to forgive the friend's $200 debt. On June 7, the seller called the neighbor to ask her where his first $20 installment payment was, and she told him at that time of her agreement with her friend. The friend has made none of the installment payments. If the seller files suit against the friend demanding payment, who will prevail? A) The seller, because he was a third-party beneficiary of the agreement between the neighbor and her friend B) The seller, because he is an assignee of the neighbor's right against her friend C) The friend, because there was no consideration for her promise to the neighbor D) The friend, because the surety provision of the SOF prevents the seller from enforcing the friend's promise

The neighbor has delegated her duties under the agreement with the seller to her friend, and the friend has agreed to assume the duties by agreeing to make the installment payments to the seller. Where a delegate's promise to perform the delegated duty is supported by consideration, there results a third-party beneficiary situation, so that the nondelegating party to the contract can compel performance or bring suit for nonperformance. The friend's promise to make the payments to the seller, totaling $180, was given in exchange for the neighbor's promise to forgive the $200 debt owed by the friend to her. The neighbor thus relinquished her right to take action against her friend for the full amount owed, thereby incurring legal detriment. Consequently, the promise of the friend was supported by consideration, and a situation arose in which the seller became a third-party beneficiary of the agreement between the neighbor and her friend, and able to enforce performance of the friend's promise to pay. (C) is incorrect because, as explained above, the friend's promise to the neighbor was supported by consideration. (B) is incorrect because there was no assignment of the neighbor's rights as against the friend; that is, the neighbor did not manifest an intent to transfer to the seller her rights against her friend. Rather, the neighbor transferred to her friend the duties that she owed to the seller. (D) is incorrect because the surety provision of the Statute of Frauds requires only that a promise to answer for the debt or default of another be evidenced by a writing. Such a promise must be collateral to another person's promise to pay rather than a primary promise (a promise to pay directly for the benefits given to another). Here, the friend did not promise the seller that if he sold the television to the neighbor and the neighbor did not pay, she (the friend) would pay. Instead, the friend promised the neighbor that she would directly perform the neighbor's obligation to pay the seller. Thus, this is not the type of promise required by the Statute of Frauds to be evidenced by a writing.

An elderly woman regularly corresponded with her only niece (her sister's daughter), who lived out of town. One day she sent her niece a letter telling her that she planned to leave everything she owned to the niece upon her death. When the woman died, her will left her entire estate valued at $200,000 to her nephew (her niece's only brother). The nephew wrote his sister a letter telling her that he felt bad about being the only person named in their aunt's will, and added, "I'm going to share her estate with you. We can discuss the details at the funeral." The niece spent $800 on a round-trip ticket to attend her aunt's funeral. After the funeral, she spoke with her brother, who told her that he had changed his mind about sharing their aunt's estate with her. He went on to say that he would be willing to share the estate with her if she were willing to share their mother's estate with him when their mother passed on. The niece responded by telling him that their mother had already signed over all the property to her. He replied that, given her attitude, he would keep their aunt's estate for himself. Later, after the two had returned to their respective homes, no longer on speaking terms, the niece sued the nephew for a 50% share of their aunt's estate. What amount, if any, should the niece realize from her suit? A) Nothing, because the aunt's will left everything to the nephew, and the nephew's letter is an insufficient basis to compel him to share B) 800, because this represents the niece's actual expenses incurred in reliance on the nephew's letter C) 100K (half of the aunt's estate), because the nephew promised her that in a signed writing D) 100K, but only if she shares their mother's estate with the nephew

The niece should not recover in her suit because there is no enforceable promise (i.e., no contract) between the nephew and the niece. Generally, a contract will not be enforced unless consideration has passed between the parties. Consideration is defined as a bargained-for exchange of a benefit to the promisor or a detriment to the promisee. Here, the nephew offered the niece half of their aunt's estate, which is certainly a detriment to him, but the niece offered nothing in return. While the nephew told the niece that they would discuss the details at the funeral, the niece's purchasing a ticket to attend the funeral is not a bargained-for detriment to her (i.e., it is not the price of the exchange) because it does not appear that the nephew's motive for the promise was to induce the niece to come to the funeral. Rather, the nephew's offer was simply to make a gift. Thus, his offer was not an enforceable promise. (B) is incorrect because the niece would be able to recover her reliance damages only under a promissory estoppel theory, and there are no grounds for promissory estoppel here. Under promissory estoppel, a promise is enforceable, at least to the extent necessary to prevent injustice, even though there was no consideration for it, if the promisor should reasonably expect to induce action by his promise and that action is in fact induced. Here, the nephew did not promise to give the niece half of their aunt's estate if she came to the funeral; rather, he only promised to share the estate and said they would talk about it at their aunt's funeral. It is not reasonably foreseeable that, based on the promise, the niece would make a special trip to attend the funeral. Indeed, it is not clear that the niece was induced to go to the funeral by the promise; she may have been planning to attend in any case. (C) is incorrect because, as explained above, the nephew's letter was an offer to make a gift; without consideration for that offer, it is unenforceable. (D) is incorrect because it relies on the existence of a contract between the nephew and the niece (he will share their aunt's estate with her in exchange for her sharing their mother's estate with him), and there is no such contract here. The nephew certainly made an offer to make such a contract, but the niece probably rejected the offer by saying that their mother's property was already hers. Even if the niece's statement was not sufficient to amount to a rejection, the nephew's reply was certainly a revocation. Thus, a contract was not formed by the exchange, and she cannot recover from the nephew on that basis.

To provide low-cost housing to the unemployed, a city has a policy of leasing empty city-owned buildings to social agencies that promise to convert or rehabilitate the buildings into habitable, low-cost apartments and to pay the city 10% of any net profit made from rentals. A church entered into such an agreement with the city and converted one of the city's abandoned office buildings into 50 small, low-cost apartments. The lease agreement used by the church provides, among other things, that the lessee must affirm a belief in God. The lease agreement was submitted to the city for approval prior to its use by the church, and it was approved. On the first day that the church made the apartments available for rent, the plaintiff, an avowed atheist, applied to lease a unit. The plaintiff's application was denied for the sole reason that the plaintiff refused to affirm a belief in God. If the plaintiff brings suit against the church on the ground that the required affirmation of a belief in God violates the plaintiff's constitutional rights, who likely will prevail? A) The plaintiff, because denial of a lease to atheists has been held to hinder the free exercise of religion B) The plaintiff, because the purpose and effect of the church's policy results in a violation of the Establishment Clause C) The church, because freedom of religion is not protected against acts of private individuals or groups or a private institution D) The church, because as an atheist, the plaintiff has not standing to challenge the lease requirement on religious grounds

The plaintiff will likely prevail because the policy violates the Establishment Clause. The First Amendment prohibits laws respecting the establishment of religion. Governmental action that does not contain a sect preference will pass muster under the Establishment Clause if it has a secular purpose, its primary effect neither advances nor inhibits religion, and it does not require excessive government entanglement with religion. The church's action will be considered to be state action here because of the significant involvement between the church and the city. (The city is leasing the building to the church, the church shares profits with the city, and the church submitted its lease forms to the city for approval.) Requiring a lessee to affirm a belief in God appears to have no secular purpose. Moreover, its primary effect probably is to advance religion. Therefore, the church's action will be found to have violated the Establishment Clause. (A) is wrong because there simply is no such Supreme Court holding. (C) is wrong because, as was pointed out above, state action can be found because of the city's significant involvement in the apartment building at issue. (D) is wrong because a person asserting a violation of the Establishment Clause does not have to allege infringement of a particular religious freedom in order to have standing; it is enough that the person is directly affected by the government action challenged.

A landowner validly conveyed a parcel of land to a veterinarian "for so long as the property is used as a veterinary practice, but if the property is used for any other purpose, it is to go to the American Cancer Society." Two years later, the landowner died, validly devising all of his property to his friend. The landowner's only heir is his daughter. Although this jurisdiction is a common law jurisdiction with respect to all real property considerations, the state's probate laws provide that future interests or estates in real property may be passed by will or descent in the same manner as present or possessory interests. Last month, the veterinarian approached the daughter and asked her to join with him to sell the parcel of land, which he had been using as an animal shelter, in fee simple absolute to a developer. The veterinarian and the daughter entered into a contract of sale with the developer. However, after consultation with an attorney, the veterinarian decided against the sale. The developer sued the daughter and the veterinarian for specific performance. Will the requested relief likely be granted? A) No, because the ACS did not join B) No, because the friend did not join in the contract of sale C) Yes, because the veterinarian had the power to sell his interest D) Yes, because together, the daughter's and the veterinarian's interest would merge and they would have a fee simple estate

The requested relief will be denied because the friend did not join in the contract of sale. The conveyance purported to create a fee simple determinable in the veterinarian subject to an executory interest in the American Cancer Society. A fee simple determinable subject to an executory interest is an estate that, on the happening of a stated event, is automatically divested in favor of a third person, who holds the executory interest. However, the executory interest in the American Cancer Society is void under the Rule Against Perpetuities because it might vest beyond lives in being plus 21 years. The charity-to-charity exception to the Rule does not apply because the veterinarian is not a charitable organization. Because any interest that violates the Rule is void and stricken from the instrument, what is left is a fee simple determinable in the veterinarian and a possibility of reverter in the landowner. On the landowner's death, the possibility of reverter passed to the friend. (A) is incorrect because the American Cancer Society does not have any interest in the land. (C) is incorrect because, although the veterinarian had the power to sell his interest, he did not own a fee simple absolute. (D) is incorrect because the daughter does not have any interest in the land. Even if she did, the merger doctrine-which provides that whenever the same person acquires all of the existing interests in land, present and future, a merger occurs and that person holds a fee simple absolute-would not apply because the friend also has an interest in the land.

Congress passed a law allowing widespread oil exploration on federal lands in the western United States. A large deposit of oil sand was discovered in one western state and Congress authorized an oil sand refining plant to be built on federal park land within the state. The refinery was built in compliance with federal pollution regulations. Pursuant to state law, the plant manager allowed the state to inspect the plant before putting it into operation. Because state refinery standards were more strict than the federal standards (in order to better protect state citizens from pollution associated with refineries), the refinery did not pass the inspection, and the state inspector refused to give the manager a permit to run the refinery. The refinery manager nevertheless began to run the refinery and was fined by the state. Which of the following is the manager's best defense against imposition of the fine? A) The state does not have a compelling interest in regulating the refinery, because it is within a federal park B) The state regulation is invalid because Congress has preempted the field of pollution control C) The state pollution regulation is invalid because it is inconsistent with the state's compelling interest in providing jobs D) The state law violates the principles of intergovernmental immunity as applied to the manager

The state law violates intergovernmental immunities principles. The states have no power to regulate the activities of the federal government unless Congress consents to the regulation. Thus, instrumentalities and agents of the federal government are immune from state regulations that interfere with their federal functions. Here, the regulation clearly interferes with the manager's duties to run the refinery. While it might be argued that the manager agreed to comply with the state regulations, because he allowed the state inspection, nothing indicates that Congress consented, and so the state regulation cannot be applied to the manager. (A) is factually incorrect; a state may still have a compelling interest in activities on federal lands, and the interest here of preventing pollution that may spread beyond federal lands probably is compelling. Nevertheless, the argument is without merit because it is irrelevant whether the state has a compelling interest in regulating federal activities; a state simply is not allowed to interfere with federal activities. (B) is incorrect because not enough facts are given to make this determination. The Supremacy Clause prohibits states from adopting laws that interfere with federal laws, and this prohibition extends to any law-even one that seeks to support the federal scheme-when Congress has preempted the field. In determining whether a field has been preempted, courts will consider the comprehensiveness of the federal scheme. The question does not give enough facts to make the determination here. (C) is incorrect because it is irrelevant. The Constitution does not require the state to favor one compelling interest over another, and so states are free to adopt laws that interfere or are inconsistent with other state goals, unless the laws are arbitrary, in which case they would violate substantive due process. The laws here do not appear to be arbitrary, and so (C) does not offer a viable defense.

A state statute provides: "Any merchant desiring to sell within this state any product or goods manufactured outside of the United States must (i) obtain a special license from the state for $50 and (ii) clearly mark the goods as to specify their country of origin." The statute makes it a misdemeanor for any merchant to willfully sell goods without complying with these statutory requirements. Which of the following statements is correct regarding the constitutionality of the statute? A) The portion of the statute requiring license fees can be sustained on the ground that reasonable inspection fees are proper; but the balance of the statute is invalid B) The portion of the statute requiring that the goods be labeled as to the country of origin can be sustained because it only requires disclosure; but the balance of the statute is invalid C) The statute is constitutionally valid as long as the burden on foreign commerce is minimal and is justified by legitimate state interests D) The statute is unconstitutional in its entirety

The statute is an unconstitutional violation of the Commerce Clause. Regulation of foreign commerce is exclusively a federal power because of the need for the federal government to speak with one voice when regulating commercial relations with foreign governments. The existence of legitimate state interests underlying state legislation will not justify state regulation of foreign commerce. The state statute, in imposing requirements for a license costing $50 and for a clear marking of goods as being from a foreign country, clearly is an attempt by the state to restrict or even eliminate the flow of such goods in foreign commerce. Thus, the statute is unconstitutional. (A) is incorrect because even if the $50 fee represents a reasonable inspection fee, the fee would still constitute an interference with foreign commerce. In addition, the facts do not indicate that the license fee has anything to do with inspection, or that the amount of the fee bears any relation to legitimate inspection purposes. (B) is incorrect because the labeling requirement imposes a burden on goods that flow in the stream of foreign commerce. Although this burden may be relatively small, it is still impermissible in light of the exclusive power held in this area by the federal government. (C) is incorrect because it states factors that would be relevant in a matter involving regulation of interstate commerce, rather than foreign commerce. Congress's power over interstate commerce is shared with the states, so that a state law may regulate local aspects of interstate commerce if it does not discriminate against out-of-state competition to benefit local economic interests and its incidental burden on interstate commerce does not outweigh the legitimate local benefits arising therefrom. However, Congress's power over foreign commerce is exclusive, so that factors such as a minimal burden on foreign commerce and the presence of legitimate state interests will not save a state law from a challenge based on the power to regulate foreign commerce

A state requires that persons holding a state license to practice a particular profession reside in the state for at least one year before engaging in that practice. A practitioner who held the state license moved into the state and shortly thereafter contracted with a local business to provide professional services. As soon as he began practicing his profession, the state licensing board sought to sanction him for violating the one-year waiting period. Which of the following provides the strongest basis for the practitioner to challenge the waiting period? A) The P&I Clause of Article IV B) The Contract Clause C) The P&I Clause of 14th A D) The Commerce Clause

The strongest basis for challenging the waiting period is the Privileges or Immunities Clause of the Fourteenth Amendment. The Fourteenth Amendment Privileges or Immunities Clause prohibits states from denying their citizens the privileges or immunities of national citizenship, including the right to interstate travel. The right to travel includes the right of newly arrived citizens of a state to enjoy the same privileges and immunities as are enjoyed by other citizens of the state. In Saenz v. Roe (1999), the Supreme Court held that the Clause was violated by a state statute limiting the welfare benefits of those who had resided in the state for less than a year. Hence, that Clause is an appropriate basis for challenging the provision. (A) is incorrect because the Privileges and Immunities Clause of Article IV prohibits discrimination against nonresidents, and here, the discrimination is against a new resident rather than a nonresident. (B) is incorrect because the Contract Clause prohibits states from enacting any law that retroactively impairs contract rights. It does not affect contracts not yet entered into, and here the state law was in place before the practitioner contracted with the local business. (D) is incorrect because the Commerce Clause generally is not violated by licensure requirements. There is an exception to the Commerce Clause (which prohibits state discrimination against interstate commerce) for state action that furthers an important, noneconomic state interest (e.g., health or safety). Generally, state professional licensing laws fall within the exception.

A student and a few of his friends were making their way to spring break. Along the way, the old van that they were driving broke down. Not wanting to miss any part of spring break festivities, the student asked the mechanic on duty at the repair shop for a rush job. The mechanic provided the student with a repair estimate, and the student, on the basis of the estimate, authorized the repair and promised to pay when he came back to pick up the van. When the mechanic called the student to tell him that the van was repaired, the student, rather than paying for the repair, told one of his friends that the mechanic had agreed to finance the repair charges and that the only thing left to do was pick up the van in the garage's parking lot. The student handed the friend a key to the van and told him to go pick the van up so that they could continue their trip to spring break. The friend did so. The mechanic makes a criminal complaint against the student for larceny of the van. If the case is prosecuted, will the student likely be found guilty? A) No, because it was the student's van to begin with B) No, because the friend took the van C) Yes, because the friend took the van from the mechanic without the mechanic's knowledge or permission D) Yes, because the student promised to pay the mechanic for his work when he came to get the van

The student will most likely be found guilty. Larceny is the taking and carrying away of the personal property of "another" with the intent to permanently deprive the other person of the property. It is possible to commit larceny of your own property if another person, such as a bailee, has a superior right to possession of the property at that time. Because the mechanic had a right to possession of the van until he was paid, the student committed larceny when he had his friend take the van without the mechanic's consent. (B) is wrong because a person can be guilty even though he did not personally engage in the behavior if he acts through an innocent agent. (D) is also incorrect. The student is guilty, but not for the reason stated in (D). He would be guilty even if he had not made the promise to pay for the van; he incurred an obligation to pay by having the repairs done.

In a drug raid, police in a city searched 25 apartments selected at random in a 300-unit housing project. In a class action approved by the federal court, the 25 tenants sued the city for violation of their constitutional rights. The tenant named as class representative gave notice to all unnamed class members, including another tenant who decided not to opt out. The class action was then certified as a "common question" type. After negotiating with the class representative, the city police agreed to pay each tenant $500 and to conduct no further raids without proper warrants. The representative and the city signed a settlement agreement and a stipulation of dismissal of the class action. The other tenant objects to the amount of damages he is to receive and would rather opt out now and proceed on his own. May the tenant opt out now? A) Yes, because a class member of a "common question" class action always has the right to opt out B) Yes, if the court allows the tenant a second opportunity to opt out C) No, because the tenant did not opt out after receiving notice of the class action D) No, if the class certification was proper

The tenant may opt out if the court allows class members a second opportunity to opt out. In a "common question" class action, a judge may refuse to approve a settlement of a class action unless the class members are given a second opportunity to opt out. [Fed. R. Civ. P. 23(e)] (A) is incorrect because there are only two opt-out periods, with the initial notice and with the settlement agreement, and the latter only applies if the judge requires it. (C) is incorrect and reflects a prior version of Rule 23. (D) is incorrect for the reasons stated above.

The plaintiff sued the defendant, who had constructed the plaintiff's house, for breach of warranty of habitability. At trial, in cross-examination of the plaintiff, the defendant's attorney asked whether the plaintiff had sued another contractor 30 years earlier, claiming similar defects in another house built for the plaintiff. The question was not objected to and the plaintiff answered that she had had some "water problems" with the first house she ever purchased, but no suit was filed. The defendant then called as a witness the contractor of 30 years earlier to testify that the plaintiff had brought suit against him for defects in the earlier house, many of which were like those now claimed to be found in the home the defendant built, but that the case was settled without trial. Should the trial court rule that the witness's offered testimony is admissible? A) Yes, as proper impeachment because the plaintiff will have an opportunity to explain or deny the witness's statement B) Yes, because the plaintiff failed to object to the defendant's questions on cross-examination relative to the prior suit C) No, because the best evidence of the former suit is the court record D) No, because its probative value is substantially outweighed by the danger that it will confuse the issues and waste time

The trial court should rule the witness's testimony inadmissible because its probative value is substantially outweighed by the danger that it will confuse the issues and waste time. Where a witness makes a statement not directly relevant to the issues in the case, the rule against impeachment (other than by cross-examination) on a collateral matter applies to bar the opponent from proving the statement untrue either by extrinsic contradictory facts or by a prior inconsistent statement. The purpose of the rule is to avoid the possibility of unfair surprise, confusion of issues, and undue consumption of time. An issue is considered collateral if it would not be admissible other than to contradict the testimony. Evidence that a person has previously filed similar claims is generally inadmissible to show the invalidity of the present claim. At best, this evidence shows the plaintiff's tendency toward litigation. Unless there is evidence that the previous claim was false, the probative value of such evidence is deemed outweighed by the risk of confusion of the issues. Because the prior suit would not be the subject of proof independent of impeachment, it is a collateral matter, and extrinsic evidence, such as the witness's testimony, is inadmissible. (A) is wrong because the testimony is not proper impeachment and is inadmissible. Further, this choice states the foundational requirement for introducing a prior inconsistent statement. This is not a prior inconsistent statement and, if the testimony were admissible, the opportunity to explain or deny would not be required. (B) is wrong because the failure to object merely meant that the plaintiff's answer to the question was admitted into evidence; it does not change the fact that the matter is collateral. Because it is a collateral matter, extrinsic evidence will not be permitted. (C) is wrong for two reasons: First, the suit is a fact that exists independently of the court record, and thus the best evidence rule would not apply. Further, as stated above, extrinsic evidence of any kind is not admissible on a collateral matter; the defendant is limited to cross-examination for impeachment in these circumstances.

A buyer for a chain of shoe stores ordered 1,000 pairs of shoes from a shoe manufacturer. The shoes cost $50 per pair, so the total contract price was $50,000. It happened that the manufacturer owed $50,000 to a trucking company. The manufacturer assigned, in writing, "all proceeds from the contract with the buyer" to the trucking company. The manufacturer notified the buyer that he had assigned the proceeds of the contract to the trucking company and then shipped the 1,000 pairs of shoes to the buyer. Upon receipt of the shoes, the buyer discovered that 10% of the shoes were defective. He sent a check for 90% of the contract price ($45,000) to the manufacturer, who deposited the check. Shortly thereafter, the manufacturer closed down its business and disappeared without a trace. The trucking company, meanwhile, demanded payment from the buyer, to no avail. If the trucking company sues the buyer for the $45,000 that the buyer paid on the contract, will the trucking company prevail? A) Yes, because the buyer had notice from the manufacturer that the contract had been assigned to the trucking company B) No, because the manufacturer wrongfully took the money that was assigned to the trucking company and is solely liable to the trucking company C) No, because the buyer fulfilled his obligations under the contract by paying the manufacturer D) No, because the trucking company could not have performed the other side of the contract by furnishing the shoes

The trucking company will be able to recover the $45,000 from the buyer because the buyer had notice of the assignment. Most contract rights may be assigned, and the right assigned here (to receive money) falls within the general rule. Once the assignment is effective, the assignee (the trucking company) becomes the real party in interest, and he alone is entitled to performance under the contract. (The assignor has been replaced by the assignee.) Once the obligor (the buyer) has knowledge of the assignment, he is bound to render performance to the assignee. Here, the assignment was effective as soon as the assignor (the manufacturer) manifested his intent that the right should be assigned (i.e., in his written assignment to the trucking company). The buyer was given notice of the assignment and, thus, was bound to pay the trucking company. The buyer breached his duty by paying the manufacturer instead of the trucking company. Thus, the trucking company may recover from the buyer for his failure to perform. (B) is wrong because the assignee, as the real party in interest, may enforce its rights against the obligor directly. (C) is wrong because, as stated above, once the buyer had notice of the assignment, he owed the duty to pay to the trucking company (the assignee), and payment to any third party, even the manufacturer (the assignor), does not discharge this duty. (D) is wrong because it is irrelevant whether the assignee could perform under the contract; the relevant question is whether the assignor could and did properly perform.

A worker was injured when a machine he was using on the job malfunctioned. The worker brought a federal diversity action against both the machine's manufacturer and the company responsible for the machine's maintenance. At trial, the worker submitted a proposed jury instruction on negligence. The court did not accept the proposed instruction and instead gave a negligence instruction that the worker's attorney believed was less favorable and legally incorrect. The attorney did not object to the negligence instruction before it was given. The jury returned a verdict for the defendants. The worker has moved for a new trial on the ground that the court's negligence instruction was improper. What argument has the best chance of persuading the court to grant the motion? A) Issues of law can be raised at any time B) The court's negligence instruction was incorrect and the worker's objection to it was preserved when he submitted his proposed negligence instruction C) The court's negligence instruction was plain error that affected the worker's substantial rights D) The need for a formal objection to a judicial ruling in order to preserve an argument has been eliminated in FRCP

y consider a plain error in the jury instructions that has not been preserved by an objection if the error affects a substantial right. This represents the only method to have the jury instruction reviewed. (A) is an incorrect statement of the law. Objection and preservation of the objection at trial is required. (B) is incorrect. Federal Rule of Civil Procedure 51 requires a proper objection on the record to preserve an objection to a jury instruction. Here, there was no objection on the record. (D) is an incorrect factual statement. Rule 51 specifically requires an objection on the record.


Conjuntos de estudio relacionados

Superficial back and scapular muscles

View Set

Ch 22: Management of Patients with Upper Respiratory Tract Disorders (3)

View Set

Intro to Bus Final Exam Practice

View Set